You are on page 1of 83

AP Biology Practice Exam

Section I: Multiple-Choice Questions


Complete one hundred multiple-choice questions in 80 minutes, worth
60 percent of the total score.

BIOLOGY
SECTION I
Time80 minutes
100 Questions
Directions: Each of the questions or incomplete statements below is followed by five suggested answers or completions. Select the one that is
best in each case.
1. During the evolutionary process, which of the following is the correct sequence of events?
A. Change in phenotype
change in genotype
speciation
selection
B.

Speciationchange in genotypeselectionchange in phenotype

C. Speciationselectionchange in phenotypechange in genotype


D. Change in genotype
change in phenotype
selection
speciation
E. Selectionspeciationchange in phenotypechange in genotype
2. A lenticel is to a plant as a _______ is to an insect.
A. stoma
B. spiracle
C. trachea
D. mouth
E. siphon

AP Biology

Questions 36 refer to the following diagram.

3. Produce digestive enzymes.


A. I, III, and V
B. II, III, and V
C. III, IV, V, and VIII
D. I, III, V, and VII
E. II, III, and IV
4. Produce(s) glucagon.
A. II and IV
B. III
C. V
D. VIII
E. II and V

AP BIOLOGY PRACTICE EXAM

5. Absorbs lipids.
A. III
B. VII
C. VI
D. IV
E. VIII
6. Produce(s) chyme.
A. VIII
B. IV
C. VII and VIII
D. II and IV
E. III
7. A new biological structure has been discovered. It is larger than
most bacteria. Although primarily composed of protein, it is covered with a lipid-based membrane. Its genetic material is double-
stranded DNA, but the structure lacks RNA. The interior of the
structure has ribosomes and a few enzymes. What is this?
A. A virus
B. A protozoan
C. An alga
D. A bacterium
E. A prion

AP Biology

8. Dinitrophenol (DNP) is a compound that allows hydrogen ions to


leak across a membrane. This leakage prevents the maintenance of
an H+ gradient. How would this impact a cell?
A. It would permit a greater growth rate, since the cell would no
longer have to spend energy pumping H+ across the membrane.
B. It would permit a greater growth rate, because the equalization
of H+ on both sides of a membrane would help the cell cope
with an acidic environment.
C. If the cell were a neuron, it would have no impact, because the
charge differential on the surface of the neuron membrane is
based on Na+ and K+, not on H+.
D. The cell would lose the bulk of its energy production because
of DNPs interference with chemiosmosis.
E. It would slow the growth rate slightly, but the cell would compensate by maintaining the function of proton pumps.
9. If you were to take 1.0 ml of a solution of HCl with a pH of 4.0
and add it to 9.0 ml of distilled water, what would be the pH of the
final solution?
A. The pH would remain unchanged.
B. The pH would rise to 5.5.
C. The pH would rise to 5.0.
D. The pH would be unmeasurable due to the amount of dilution.
E. The pH would rise to 7.0.
10. From where do the electrons used in photosynthesis originate?
A. Glucose
B. NAD
C. Photons
D. Chlorophyll
E. Water

AP BIOLOGY PRACTICE EXAM

11. What distinguishes mammalian from plant cell membranes?


A. Plant membranes contain chlorophyll, while mammalian membranes do not.
B. Mammalian membranes contain cholesterol, while plant membranes do not.
C. Plant membranes contain ion channels, while mammalian
membranes do not.
D. Mammalian membranes are attached to a cytoskeleton, while
plant cells are not.
E. Plant membranes contain cellulose, while mammalian cell
membranes do not.
12. In fungi, sporangiospores are found in bag-like structures. Ascospores
are also found in bag-like structures, but are always present in even
numbers. Why are these spore types not the same?
A. They are basically the same, but the sporangiospores are always
larger.
B. Ascospores are formed as a result of meiosis, while sporangiospores are a result of mitosis.
C. Sporangiospores are produced by sexual recombination, while
ascospores are produced asexually.
D. They are basically the same, but ascospores are always larger.
E. The structures are the same, but they were named differently
before they were determined to have the same function.

AP Biology

13. One line of evidence for the evolutionary process is the identification of homologous structures. Of the following pairs of characteristics, which could be used in support of this mechanism?
A. The wings of birds compared to the wings of insects
B. The eyes of insects compared to the eyes of humans
C. The cardiovascular system of animals compared to the vascular
system of plants
D. The wings of birds compared to the legs of bats
E. The fins of whales compared to the hooves of horses
14. If a person contracts and survives an infectious disease such as measles, they tend not to get the disease again later in life. What cells are
involved in this protective process?
A. Platelets
B. Neurons
C. Lymphocytes
D. Neutrophils
E. Erythrocytes
Questions 1517 refer to the following simplified diagram.

AP BIOLOGY PRACTICE EXAM

15. The diagram represents a central process of catabolism. Identify the


process designated as (I).
A.

Electron-transport chain

B.

Light-independent reactions

C. TCA cycle
D. Glycolysis
E. Transcription
16. What element or compound is needed for the process designated as
(III) to occur?
A. Chlorine
B. Nitrogen
C. Oxygen
D. Potassium
E. Sodium
17. Which combination of processes takes place entirely within a cellular organelle?
A. I only
B. III only
C. I and II
D. II and III
E. I, II, and III

AP Biology

18. Animal phyla are broadly categorized into acoelomates, deuterostomates, protostomates, and pseudocoelomates. Identify the phyla
that are classified as protostomates.
A. Platyhelminthes and cnidarians
B. Echinodermatans and chordates
C. Molluscans and arthropods
D. Nematodes
E. Poriferans
19. Some dinosaur bones were discovered fairly recently that still contained significant amounts of the protein collagen in a non-fossilized
form. Enough material was available to determine its amino-acid
sequence. This allowed researchers to compare its sequence with the
sequences of collagen from currently existing animals. What group
of organisms did the dinosaur sequence most closely resemble?
A. Birds
B. Lizards
C. Amphibians
D. Rodents
E. Fish
20. A cell is composed of thousands of molecules that range in size from
very small to very large. Select the sequence of molecular size from
the smallest to the largest.
A.

Carbon dioxidephospholipidDNA polymeraseribosome

B.

PhospholipidDNA polymeraseribosomecarbon dioxide

C.

Phospholipidcarbon dioxideribosomeDNA polymerase

D. Carbon dioxideDNA polymerasephospholipidribosome


E.

DNA polymeraseribosomecarbon dioxidephospholipid

AP BIOLOGY PRACTICE EXAM

Question 21 refers to the following graph.

21. A researcher was analyzing the life expectancy of a rodent found


on an island off the coast of California. She was able to date its age
based on the wear on its teeth. Her data is presented in the graph.
What kind of survivorship curve would this data represent?
A. A type I survivorship pattern
B. A blend of a type I and a type III survivorship pattern
C. A blend of a type II and a type III survivorship pattern
D. A type III survivorship pattern
E. A type II survivorship pattern

10

AP Biology

Questions 22 and 23 refer to the narrative below and the options that
follow.
When restriction fragments produced from the same
endonuclease are mixed under the proper conditions,
they will spontaneously reanneal. However, they will not
function properly within a cell until the nicks in the strands
are repaired.
A. Taq DNA polymerase
B. Reverse transcriptase
C. DNA-dependent RNA polymerase
D. DNA ligase
E. Eco RI
22. Identify a restriction endonuclease.
23. What enzyme is necessary to repair the nicks?
24. Which relationship presents a condition that could be identified as
exploitative competition?
A. A woodpecker and a wren
B. A sturgeon and a minnow
C. A mouse and a mole
D. A lichen and a rock
E. Mistletoe and a tree

AP BIOLOGY PRACTICE EXAM

11

25. Inside which portion of a cell does translation take place?


A. The endoplasmic reticulum
B. The nucleus
C. The cytosol
D. The Golgi complex
E. The cell membrane
26. A woman has a colorblind father. Her son also has a colorblind
father. Colorblindness is caused by an X-linked recessive gene. If
only male offspring are considered in this case, what is the probability that her son will be colorblind?
A. 100 percent
B. 50 percent
C. 0 percent
D. 25 percent
E. 75 percent
27. What is the best way to describe the steps involved in converting
fatty acids to acetyl-CoA?
A. They are catabolic, because the free energy increases as a result
of the process.
B. They are metabolic, because the free energy increases as a result
of the process.
C. They are anabolic, because the free energy decreases as a result
of the process.
D. They are catabolic, because the free energy decreases as a result
of the process.
E. They are anabolic, because the free energy increases as a result
of the process.

12

AP Biology

Questions 2830 refer to the diagram and the options that follow.

A. I, II, III, V, and VI


B. I, V, and VI
C. VI
D. II and III
E. IV and VI
28. Require or utilize both oxygen and carbon dioxide.
29. Classified as dermal tissue or structures.
30. Closely associated with transpiration.

13

AP BIOLOGY PRACTICE EXAM

31. Mast cells are leukocytes contained in tissues. They are filled with
granules that contain histamine. These cells are involved in allergies,
including hay fever, for which antihistamines are used as therapy.
What mechanism is the cause of the allergic reaction?
A. When an allergen binds to specific receptors on a mast cell, it
induces cellular degranulation and the release of histamine into
the tissues.
B. Seasonal allergens irritate the nasal mucosa and cause cellular
infiltration by mast cells.
C. Mast

cells

phagocytose

allergens

that

induce

cellular

proliferation.
D. Mast cells rove through tissues removing both histamines and
antihistamines that may be causing the allergy.
E. Antihistamines are antibodies that bind to histamine to produce a complex, which is then removed from the tissue by
mast cells.
32. The genome of the rabies virus is negative-sense RNA. Negative
sense means that the nucleic acid sequence is complementary to
mRNA. What enzyme must be present in order for this virus to
produce its viral proteins and replicate?
A. DNA-dependent DNA polymerase
B. RNA-dependent RNA polymerase
C. DNA ligase
D. DNA-dependent RNA polymerase
E. Restriction endonuclease

14

AP Biology

Questions 3336 refer to the graph and narrative that follow.

The options below apply only to questions 33 and 34. The


normal blood glucose level is 70150 mg/dl. The arrow indicates
an injection of insulin only in someone who is diabetic.
A. I and II
B. III
C. I, II, and IV
D. IV
E. II
33. Which curve(s) indicate(s) probable diabetes?
34. Which curve(s) indicate(s) diabetic shock?

AP BIOLOGY PRACTICE EXAM

15

35. To what do you attribute the drop, then rise, of blood glucose levels
seen in curve II?
A. Very poor monitoring of blood glucose levels
B. Flaws in the instrumentation or methods used to determine the
blood glucose levels
C. Ingestion of additional glucose at about time 2 hours
D. A second injection of insulin at about time 2 hours
E. Sporadic release of insulin by the pancreas
36. The normal range is provided as 70150 mg/dl. What would be the
equivalent expressed as g/l?
A.

0.71.5 g/l

B.

7.0150 g/l

C.

0.0070.015 g/l

D. 0.070.15 g/l
E.

0.00070.0015 g/l

16

AP Biology

Questions 37 and 38 refer to the following narrative.


Erythroblastosis fetalis is a condition that affects a fetus in
utero caused by an Rh incompatability with the mother.

37. What is the mechanism that produces this condition?


A. Antibodies from the fetus attack antibodies from the mother
found in fetal circulation.
B. Antibodies from the fetus attach to the red blood cells within
the maternal circulation.
C. Lymphocytes from the mother cross the placenta and attack
the red blood cells of the fetus.
D. Lymphocytes from the fetus cross the placenta and attack the
red blood cells in the mother.
E. Antibodies from the mother cross the placenta and attack the
red blood cells of the fetus.
38. This problem is caused only when a(n) __________ mother has a
child with phenotype _________.
A. Unsensitized Rh+; Rh
B. Sensitized Rh; Rh
C. Unsensitized Rh; Rh+
D. Unsensitized Rh; Rh
E. Sensitized Rh; Rh+

AP BIOLOGY PRACTICE EXAM

17

39. What is the relationship between glucose and sucrose?


A. Glucose is sucrose with an extra OH group at the 3 position.
B. Sucrose is composed of two glucose molecules joined by a
glycoside bond.
C. A glucose molecule joined to a fructose molecule makes a molecule of sucrose.
D. Glucose is composed of two sucrose molecules joined by a
glycoside bond.
E. Glucose is composed of a fructose and a sucrose molecule
joined by a glycoside bond.
Questions 4042 refer to the following diagram and narrative.

The lines represent three species within a single isolated


ecosystem as observed by a team of qualified researchers.

18

AP Biology

40. What relationship between two populations can you reasonably


deduce when comparing curves I and II?
A. The relationship is neutral.
B. There is strong evidence of competition but with resource
partitioning.
C. The population designated II appears to be an invasive species
that outcompetes the other.
D. The population designated I appears to be an invasive species
that could not survive in a new niche.
E. The population designated II appears to be a parasitic species
that uses the other as its host.
41. What relationship does the population designated as III reasonably
have with the other two populations?
A. The population designated III is a competitor of both of the
other populations.
B. The population designated III is a predator of population II but
not I.
C. The population designated III is a predator of population I but
not II.
D. The population designated III has a neutral relationship with
both of the other populations.
E. The population designated III is a predator of both populations
II and I.

AP BIOLOGY PRACTICE EXAM

19

42. To what might you reasonably attribute the loss of population III
within the ecosystem?
A. Population III was a prey of population I, which completely
consumed all its members within the ecosystem.
B. Population III was a prey of population II, which completely
consumed all its members within the ecosystem.
C. Population III was a predator of population II, but it could not
survive in the ecosystem.
D. Populations II and III both used population III as their prey,
completely consuming all its members within the ecosystem.
E. Population III was a predator of population I but it could not
survive in the ecosystem.
43. What differences are there between the cell membranes of eukaryotes and prokaryotes?
A. Prokaryotic cell membranes are thinner and contain more proteins.
B. Eukaryotic cell membranes contain more proteins and larger
phospholipids.
C. Gases and water pass through a prokaryotic cell membrane at
greater rates than through those of eukaryotes.
D. Many functions that are compartmentalized within organelles of
eukaryotes are associated with the cell membranes of prokaryotes.
E. There are no significant differences in the structure or function
of the cell membranes.

20

AP Biology

44. Which of the following best explains the origin of the predation
ability of Venus flytraps?
A. Only those plants with the ability to supplement the poor
nutrient content of their soils could survive.
B. Predation allowed the Venus flytrap to outcompete other species because of its ability to move rapidly.
C. As consumers instead of producers, Venus flytraps have greater
access to better nutrients.
D. Predation allowed the Venus flytrap to collect a wider variation
of preformed organic materials.
E. Better nutrients allowed the Venus flytrap to grow faster than
other plants.
45. A type of fungus exists that lives symbiotically with plants. What is
that fungus?
A. Mycorrhizae
B. Lichens
C. Mushrooms
D. Morels
E. Truffles
46. Which of the following experiments first ascertained that the
nucleus possessed the ability to control phenotype?
A. Experiments with radiolabelled phages
B. Experiments with bacteria that identified DNA polymerase
C. Transformation experiments with Streptococcus pneumoniae
D. Identification of restriction endonucleases
E. Grafting experiments with Acetabularia

AP BIOLOGY PRACTICE EXAM

21

47. A father with type B blood and mother with type A blood have a
child. Their child, while in a biology lab at school, tests her blood
and discovers she has type O blood. Does she have any concerns
about her parentage?
A. Yes, because she should have type AB blood if they are her true
biological parents.
B. No, because type O blood is possible if her parents both had
genotypes AB.
C. No, because both of her parents could be heterozygous.
D. Yes, because both of her parents might be heterozygous.
E. No, because blood types A and B are codominant.
48. The HIV genome is roughly 10,000 nucleotides in size, while
the human genome compares at about 3,000,000,000 base pairs.
Reverse transcriptase has an error rate of about 1 in 20,000 bases.
What does this imply about the evolution of the virus?
A. The evolutionary rate is rather slow, because it is unlikely that
any one virus will contain a mutation.
B. The rate of evolution is extremely high, because every infected
cell will produce viruses with mutations.
C. The rate of evolution is rather slow, because the genome is
so small.
D. The rate of evolution is extremely high, because the human
genome is so comparatively large.
E. The evolutionary rate is slow, because the vast majority of
HIV mutations will prevent the virus from binding to human
cell receptors.

22

AP Biology

Questions 4951 refer to the diagram

49. Which organ or tissue contains this structure?


A. Liver
B. Ovary
C. Lung
D. Kidney
E. CNS
50. Vasopressin affects the functioning of this structure. Which of the
following secretes this substance?
A. Posterior pituitary
B. Liver
C. Hypothalamus
D. Anterior pituitary
E. Adrenal glands

AP BIOLOGY PRACTICE EXAM

23

51. What product or process is this structure known for?


A. Production of urine
B. Production of LH and FSH
C. Feedback control of the autonomic nervous system
D. Gas exchange
E. Production of bile
52. Human cells contain DNA sequences that match the sequences
found in bacterial 16S rDNA. What is your response to this
statement?
A. The statement is untrue, because healthy human cells should
contain no bacterial DNA.
B. The statement is true, because mitochondria contain bacterial
DNA sequences.
C. The statement is untrue, because there is no such thing as
rDNA, only rRNA.
D. The statement is true, because the 16S sequences are part of
the human 18S sequences.
E. The statement is true, because there are many human sequences
that match 16S sequences.
53. The Na+/K+ pump uses a form of ________ _______ known as
_________.
A. passive transport; uniport
B. facilitated diffusion; antiport
C. active transport; symport
D. facilitated diffusion; uniport
E. active transport; antiport

24

AP Biology

54. Many mammalian cell membrane receptors associated with the


immune system contain repetitive structures known as the immunoglobulin fold. Receptors containing even one of these folds are
considered members of the immunoglobulin superfamily. How
would you explain this common feature in so many molecules?
A. All of the genes that produce these proteins used to be part of
a much larger gene that fragmented in the past.
B. All of the genes in question are retrotransposons.
C. Most of the genes in question are introns that experienced
mutations that removed the splice sequence.
D. The fold was so successful as a signaling molecule that it encouraged its propagation.
E. The fold is repetitive because of gene duplication.
Questions 55 and 56 refer to the following narrative.
Assume you had a 10 m2 pond to study. You measured the
amount of light energy bathing the water surface at 1,000 kcal.

55. About how much of that available energy would be expected to be


converted into producer biomass?
A. 100 kcal
B. 1,000 kcal
C. 2,000 kcal
D. 10 kcal
E. 200 kcal

AP BIOLOGY PRACTICE EXAM

25

56. Based on the energy captured above, about what percentage of that
would be transferred to higher trophic levels?
A. 1.65 percent
B. 16.5 percent
C. 33 percent
D. 3.3 percent
E. 66 percent
57. If a woman who suffers from rheumatoid arthritis becomes pregnant, the arthritis symptoms often subside temporarily during her
pregnancy. Not coincidentally, a woman who has once had genital
warts caused by the human papillomavirus (HPV) will find they
return temporarily during her pregnancy. What do these two observations have in common?
A. Both effects are caused by the production of FSH and LH during pregnancy.
B. Both effects are caused by the reduction of progesterone and
estrogen during pregnancy.
C. Both are caused by the partial suppression of the immune system during pregnancy.
D. Both effects are caused by the production of progesterone and
estrogen, and the drop in -HCG levels, during pregnancy.
E. Both effects are caused by the production of oxytocin during
pregnancy.

26

AP Biology

Questions 58 and 59 refer to the following diagram.

58. The diagram indicates an error in some process. What is that process?
A. Meiosis
B. Crossing over
C. Pleiotropy
D. Misogyny
E. Mitosis
59. Which of the following is true about the process as indicated?
A. It is the cause of sickle cell anemia.
B. It is the failure to get equal distribution of chromosomes during
mitosis.
C. It is associated with susceptibility to HIV infection.
D. It is an important part of the double-fertilization process.
E. It is the cause of the superfemale syndrome (XXX).
60. What do these two conditions have in common: webbed fingers and
toes in humans and the development of cancer cells?
A. Both are caused by excessive cell growth.
B. Both are caused by a loss of apoptosis.
C. Both are a result of exposure to carcinogenic compounds.
D. Both are caused by excessive fat-soluble vitamin levels.
E. Both are caused by infections from cancer-associated viruses.

AP BIOLOGY PRACTICE EXAM

27

61. Tobacco mosaic virus (TMV) is very common in the environment


and is frequently isolated from tobacco products. What health
implication does this present?
A. TMV is a cause of lung cancer.
B. TMV is associated with immune suppression.
C. TMV is associated with skin tumors.
D. There are no health implications; TMV cannot infect human
cells.
E. TMV is associated with genetic abnormalities within any cell
exposed to secondhand smoke.
Question 62 refers to the following image.

62. Which has the most phylogenetic relatedness to what is seen here?
A. Lycophytes
B. Monocots
C. Gingkos
D. Mosses
E. Ferns

28

AP Biology

63. In addition to enzymes, at least one other organic molecule is known


to be capable of catalysis. What is that molecule?
A. DNA
B. Fatty acids
C. Cellulose
D. RNA
E. ATP
64. What is the primary source of the electrons used in the mitochondria?
A. Water
B. Glucose
C. ATP
D. Cellulose
E. Oxygen
65. Large populations of organisms are centered on deep sea hydrothermal vents. Which of the following is true about these populations?
A. The energy captured by these organisms is thermal in nature
and allows them to thrive in mixed populations.
B. Only a single species is found at any one vent, although the
species varies from vent to vent.
C. The species found at each site are usually evenly mixed between
producer plants and consumer microorganisms and animals.
D. Fungi and protozoans are usually not represented in these
populations.
E. Energy from mineral ions feed producers, which in turn feed
varieties of consumers.

AP BIOLOGY PRACTICE EXAM

29

66. A certain species of bird has two feather variations displayed by


males: a dull-colored one that blends into their preferred habitat
and a brightly colored one that is very obvious. The adult females
are dull colored. The predation rate for brightly colored adults is
three times higher than for the dull-colored adults. In spite of this,
the brightly colored plumage persists in the population. How would
you explain this?
A. Females prefer the brighter plumage and so mate with this
phenotype more often.
B. The dull plumage provides a selection advantage over the
bright plumage.
C. The adults with dull plumage require less energy investment to
ensure their survival.
D. Those brightly colored birds that do survive have more highly
developed survival skills than those that do not survive.
E. Dull-colored adult males are more lethargic than the brightly
colored birds.

30

AP Biology

Questions 6769 refer to the diagram below and the options that follow.

A. VII
B. I and VI
C. II, III, IV, and V
D. II, V, and VII
E. III and VII
67. Meiosis takes place in which structure(s)?
68. If this flower relied on wind pollination, then this/these structure(s)
would be a lot more pronounced.
69. Aggregate fruit are produced from a single flower with multiple
numbers of these structures.

AP BIOLOGY PRACTICE EXAM

31

70. As has occurred innumerable times in the past, the global mean
temperature appears to be rising, at least in the short term. Which
of the following is an inevitable result of this change?
A. First frost dates will be delayed.
B. Birds will migrate shorter distances.
C. Oxygen levels in the oceans will rise.
D. Available fertile lands will decrease as desertification increases.
E. Animal life will be reduced worldwide.
71. The damming of the Nile River near Aswan in the 1960s introduced
a new disease to the area. While the water once flowed freely, stopping its flow allowed new organisms to thrive in newly opened
niches. One of these new organisms causes a disease called schistosomiasis, which infects humans as part of its life cycle. What is the
taxonomic classification of this organism?
A. Arachnid
B. Helminth
C. Cephalopod
D. Bacterium
E. Cladoceran
72. Antigen-
presenting cells communicate with effector cells of the
immune system. Which of the following mechanisms account for this?
A. Direct cell membrane-to-membrane contact
B. Quorum sensing
C. Release of highly specific steroidal hormones
D. Release of neurotransmitter-like substances
E. Induction of rapid ion flow through the membrane

32

AP Biology

Questions 7375 refer to the following narrative.


Shigella is a bacterium that is very closely related to E. coli but
is incapable of fermenting lactose (lactose negative). A gene for
resistance to the antibiotic ampicillin (ampR) was inserted in
between two of the structural genes of the lac operon and then
inserted into Shigella.

73. What would be the effect of this insertion?


A. The cell would immediately be resistant to ampicillin.
B. The cell would be resistant to ampicillin only in the presence of
tryptophan.
C. The cell would produce the enzyme -lactamase as soon as it
started to grow exponentially.
D. The cell would spontaneously utilize DNA-editing enzymes to
restore the original genotype.
E. The cell would be resistant to ampicillin only in the presence of
lactose.
74. If Shigella is normally lactose negative, from where did the lac
operon originate?
A. The bacterial genome was removed, the lac operon was added,
and then the genome was inserted back into the original cell.
B. The lac operon was present on a plasmid which was inserted
into the cell by transformation.
C. The bacterial genome was removed and replaced by a plasmid
containing the lac operon.
D. The lac operon developed in the cell by random mutation and
strong selection pressures.
E. The lac operon was derived from mutations of the tryptophan
operon which is normally present in Shigella.

33

AP BIOLOGY PRACTICE EXAM

75. What enzyme regularly used by genetic engineers would be vital for
the successful completion of this exercise?
A. DNA polymerase
B. Reverse transcriptase
C. Taq polymerase
D. -galactosidase
E. Restriction endonuclease
76. Assume that you have been hired to work in a lab that tests plant
hormones for increasing crop yield. Your job today is to test spray a
large plot with a selected hormone to determine its effect in hybrid
vigor. Which of the following would you want to avoid using?
A.

2,4-D

B. IAA
C. Gibberellin
D. Cytokinin
E. Abscisic acid
77. Place the following vessels or tissues in the proper sequence from
containing the highest concentration of CO2 to the lowest.
A.

Pulmonary

veinsleft

atriumtissuesright

ventricle

pulmonary arteries
B.

Left atriumtissuesright ventriclepulmonary veins


pulmonary arteries

C.

Pulmonary arteriesright ventricletissuesleft atrium


pulmonary veins

D. Right ventriclepulmonary arteriespulmonary veinsleft


atriumtissues
E.

Tissuespulmonary veinspulmonary arteriesleft atrium


right ventricle

34

AP Biology

78. Individuals born with a condition known as phenylketonuria must


control their intake of phenylalanine or suffer developmental problems. Because of this they must avoid foods rich in what?
A. Proteins and lipids
B. Fats and oils
C. Proteins
D. Simple sugars
E. Complex carbohydrates and oils
Questions 7980 refer to the following table and narrative.

The genes that code for the human leukocyte antigen (HLA)
antigens, also known as the major histocompatability complex
(MHC) genes, are all located on human chromosome 6. Two
brothers and a sister participated in a bone marrow donor
registration, and their HLA typing results are given above.

AP BIOLOGY PRACTICE EXAM

35

79. What is the probability that, if they had a fourth sibling, that sibling
would match one of the three above?
A. 25 percent
B. 100 percent
C. 0 percent
D. 50 percent
E. 75 percent
80. What is your explanation for the single HLA-A result for sibling 3?
A. A gene-deletion event occurred during crossing over.
B. Nondisjunction occurred during meiosis.
C. A laboratory error caused the lost data; it should have a second
result.
D. A metabolic error prevented the synthesis of the second antigen.
E. Both parents were heterozygous for HLA-A1.

36

AP Biology

Question 81 refers to the following graph.

81. Based on this monitoring of an individual infected with HIV at six


months, who was untreated during the time period indicated, how
would you identify what the three curves signify?
A. I = viral antigen in the blood; II = anti-HIV antibodies in the
blood; III = CD4+ cells in the blood
B. I = viral antigen in the blood; II = CD4+ cells in the blood; III =
anti-HIV antibodies in the blood
C. I = CD4+ cells in the blood; II = anti-HIV antibodies in the
blood; III = viral antigen in the blood
D. I = anti-HIV antibodies in the blood; II = viral antigen in the
blood; III = CD4+ cells in the blood
E. I = CD4+ cells in the blood, II = viral antigen in the blood; III =
anti-HIV antibodies in the blood

AP BIOLOGY PRACTICE EXAM

37

82. Where in a plant cell is a water molecule broken down to release


oxygen gas, electrons, and hydrogen ions?
A. In the inner matrix of the mitochondrion
B. In the outer membrane of the chloroplast
C. In the intermembrane space of the mitochondrion
D. In the thylakoid space of the chloroplast
E. In the intermembrane space of the chloroplast
83. What is the best measuring stick to use to determine the degree of
fitness in evolutionary selection?
A. The increase in lifespan resulting from selection
B. The increase in chromosomal number within the nucleus
C. Reproductive success
D. The number of available alleles for any one gene
E. The increase in geographical territory for any one species
84. Identify the general relative sizes of the listed structures from the
smallest to the largest.
I = bacterium
II = enzyme
III = nucleus
IV = chromosome
V = molecule of water
A.

VIIIIVIII

B.

IIVIVIIII

C.

VIIIIIIIV

D. IIVIIVIII
E.

IIVIVIII

38

AP Biology

Question 85 refers to the following graph.

85. The graph displays the activity curves for three human enzymes.
Identify the curve designation with the enzyme.
A. I = pepsin; II = chymotrypsin; III = fibrin
B. I = chymotrypsin; II = fibrin; III = pepsin
C. I = fibrin; II = pepsin; III = chymotrypsin
D. I = pepsin; II = fibrin; III = chymotrypsin
E. I = fibrin; II = chymotrypsin; III = pepsin

AP BIOLOGY PRACTICE EXAM

39

86. Why do toadstools form fairy rings after a rain?


A. The ring reflects a pH gradient which is highest in the center of
the circle and diffuses outward. The size of the ring varies with
the alkalinity at the center.
B. The ring appears around the remnants of a tree stump that
provides nutrients at the perimeter of the buried wood.
C. The ring usually appears at the periphery of large buried boulders or other dense material.
D. In actuality, the appearance of a fairy ring is usually just the
minds attempt to make a pattern out of random placements of
toadstools.
E. The ring indicates the intersection of a colony of subsurface
fungal hyphae expanding into a surrounding colony with a different mating type.
Questions 87 and 88 refer to the following narrative.
Hemophilia is a genetic disorder caused by an X-
linked
recessive allele. It is of historical interest because of the
extensive familial intermarriages of the royal houses of Europe
during the seventeenth to nineteenth centuries AD.

87. In evolutionary terms, why has this lethal allele continued to be


passed on?
A. Because of the founder effect
B. Because, while producing a bleeding disorder, it increases longevity
C. Because it only affects female carriers of the trait
D. Because the recessive allele provides a selection advantage
E. Because of disruptive selection

40

AP Biology

88. Which of the following best describes this form of hemophilia?


A. This disorder is caused by reduced production of thrombocytes
in the bone marrow.
B. This disorder is caused by an increased production of antithrombin.
C. This disorder is caused by an absence of factor VIII in the blood.
D. This disorder is caused by a loss of the clotting ability of thrombocytes, which are present in normal numbers in the blood.
E. This disorder is caused by an increased level of anticoagulants
in circulation and in the tissues.
89. Numerous animals, notably reptiles, are capable of reproduction
and maintaining numbers when populations are composed of only
single sex members, even in isolated areas. How do you explain
this statement?
A. It is not possible and the statement is false.
B. When only females are present, hormonal signals will cause the
dominant female to develop male characteristics and engage in
sexual reproduction with the other females.
C. Meiosis within unfertilized eggs will arrest at a diploid stage and
undergo normal fetal development in an all-female population.
D. When only males are present, random males will become hermaphroditic and lay eggs fertilized internally.
E. The sperm from males can be stored for years within a female,
allowing normal clutches of eggs to hatch years after males
have disappeared from a population.

AP BIOLOGY PRACTICE EXAM

41

90. An experiment is set up with a culture of algae undergoing normal


photosynthesis. A pulse of radioactive carbon, in the form of 14CO2,
is inserted into the culture at time zero. At five-second intervals,
some of the cells are removed by dropping them into boiling water
to halt metabolic processes. What is the first organic material that
would be identified as being radioactive during this process?
A. ATP
B. PGA
C. ADP
D. Glucose
E.

Acetyl-CoA

91. Hydrilla is a vascular invasive plant species that grows under the
surface of lakes at a very high density during the summer but dies
back to its buried rhizomes during the winter. It has been linked
to oxygen depletion of the waters in which it is found, in spite of
the fact that it is photosynthetic. How can you account for this
seeming contradiction?
A. While photosynthetic, Hydrilla consumes oxygen at a faster
rate than it releases it because of its very high rate of growth.
B. Hydrillas nutrient density is so great that fish that feed upon it
proliferate beyond the carrying capacity of the lakes in which it
is found.
C. Because Hydrilla is vascular, it binds up large quantities of
oxygen in its biomass.
D. As an aquatic plant, Hydrilla requires large quantities of
oxygen to fill its air bladders.
E. When the vegetative structures die off in the winter, decomposing bacteria consume the oxygen during their rapid growth.

42

AP Biology

Questions 9294 refer to the following graph.

92. Given that the curve designated II is FSH, identify the other three.
A. I = LH; III = progesterone; IV = estrogen
B. I = estrogen; III = LH; IV = progesterone
C. I = progesterone; III = LH; IV = estrogen
D. I = LH; III = estrogen; IV = progesterone
E. I = estrogen; III = progesterone; IV = LH
93. Identify the sources of the hormones.
A. Ovary = I and II; pituitary = III; hypothalamus = IV
B. Ovary = III and IV; pituitary = I and II
C. Ovary = I and II; pituitary = III and IV
D. Hypothalamus = I and II; pituitary = III and IV
E. Ovary = I and II; pituitary = IV; hypothalamus = III

AP BIOLOGY PRACTICE EXAM

43

94. Identify the day of follicle rupture.


A. About day 14
B. About day 10
C. About day 0
D. About day 21
E. About day 25
95. Which of the following observations support the fluid mosaic model
of membrane structure?
A. The capping of fluorescent antibodies specific for cell-surface
receptors
B. The presence of the cytoskeleton underneath the surface of the
membrane
C. The measurement of ions flowing through ion-gated channel
proteins by patch clamps
D. The pseudopodial movement of amoebas
E. The scaffolding and kinetic proteins associated with the movement of cilia
96. Brocas area, the portion of the brain associated with the motor control of speech, is localized in which portion of the brain?
A. The parietal lobe
B. The cerebellum
C. The occipital lobe
D. The frontal lobe
E. The lateral sulcus

44

AP Biology

97. Which of the following means of reproduction is the most ancient?


A. Viviparity
B. Oviparity
C. Ovoviviparity
D. Placental viviparity
E. Viviparous matrotrophy
98. When carbohydrate monomers are joined together, the process is
identified as ___________ ______________, and when they are broken apart again it is called ___________.
A. hydrolytic bonding; degradation
B. hydrogen bonding; depletion
C. condensation synthesis; hydrolysis
D. polar synthesis; hydrolysis
E. condensation synthesis; anabolism
99. Some plants have the ability to twine around objects. What is this
ability to respond to physical contact with a growth response called?
A. Seismonastic movement
B. Phototropism
C. Thigmotropism
D. Somnambulism
E. Gravitropism

AP BIOLOGY PRACTICE EXAM

45

100.
Many organisms have been used in research to help elucidate
biological processes. Which of the following is used exclusively in
developmental biology?
A. Drosophila melanogaster
B. Caenorhabditis elegans
C. Pisum sativum
D. Escherichia coli
E. Daphnia pulex
END OF SECTION I

46

AP Biology

Section II: Free-Response Questions


Complete four essay questions, with ten minutes of reading and planning time and an hour and a half of writing time. Each question is worth
10 percent of the total exam score.

BIOLOGY
SECTION II
Reading and planning time10 minutes
Writing time1 hour and 30 minutes
Directions: Answer all questions.
Your answers must be in essay form. Neither outlines alone nor diagrams
alone are sufficient. It is important that you read and understand each
question before you begin to write.
1. Cellular homeostasis must be maintained to sustain life and life processes. In order to accomplish this, the cell must possess the ability
to operate with both positive and negative control mechanisms.
A. Identify and explain ONE positive and ONE negative control
mechanism involved in gene expression.
B. Identify and explain ONE positive and ONE negative control
mechanism involved in enzyme kinetics.
2. Organismal homeostasis must be maintained to sustain life and
insure the continuation of the species. In mammals, this maintenance is often controlled by both neural and hormonal mechanisms.
A. Discuss the neural mechanism of the control of blood pressure.
B. Discuss the kidneys role in maintenance of blood pressure.
C. Identify TWO disease processes caused by failure to control
blood pressure.

AP BIOLOGY PRACTICE EXAM

47

3. Speciation results from the application of selective pressures until


two non-interbreeding populations develop.
A. Identify and explain TWO prezygotic isolating mechanisms.
B. Identify and explain TWO postzygotic isolating mechanisms.
4. Plants maintain homeostasis but lack the pumps and muscles that
animals have in order to do so.
A. Identify the elements associated with fluid flow within vascular
plants. Describe the mechanisms that produce the flow.
B. In order to maintain fluid flow, the vascular structure must be
protected. Explain TWO such protective mechanisms.
END OF EXAM

48

AP Biology

Answers to the AP Biology Practice Exam


Section I: Multiple-choice questions
Answer Key
1. D

25. C

2. B

26. B

3. A

27. D

4. C

28. D

5. B

29. B

6. E

30. E

7. A

31. A

8. D

32. B

9. C

33. C

10. E

34. D

11. B

35. C

12. B

36. A

13. E

37. E

14. C

38. E

15. D

39. C

16. C

40. C

17. D

41. B

18. C

42. C

19. A

43. D

20. A

44. A

21. E

45. A

22. E

46. E

23. D

47. C

24. A

48. B

AP BIOLOGY PRACTICE EXAM

49. D

75. E

50. A

76. A

51. A

77. C

52. B

78. C

53. E

79. E

54. E

80. E

55. D

81. B

56. B

82. D

57. C

83. C

58. A

84. A

59. E

85. D

60. B

86. E

61. D

87. A

62. C

88. C

63. D

89. C

64. B

90. B

65. E

91. E

66. A

92. D

67. E

93. B

68. C

94. A

69. D

95. A

70. A

96. D

71. B

97. B

72. A

98. C

73. E

99. C

74. B

100.B

49

50

AP Biology

Answer Explanations
1. D. This question is designed to evaluate your understanding of the
basic process of evolution.
Jean-Baptiste Lamarck proposed that physical changes that developed
in an individual would be passed on to its progeny. This could be interpreted to mean that if a person worked out a lot and became buff, that
extra toning would be passed on to his or her children. Charles Darwin,
on the other hand, proposed that selection was the process that determined what characteristics were passed on. In order for a characteristic to
appear, there must first be a mutation in the DNA (change in genotype).
Once the DNA has changed, the changed gene is expressed, modifying
what characteristic may be seen or measured (change in phenotype).
After the introduction of the new phenotype increases the diversity
within the population, environmental pressures then serve to select
which phenotype is more fit under those conditions. Accumulation of
a battery of new phenotypes eventually will cause divergence from the
original species genotype.
2.

B. The purpose of this question is to determine if you can identify

respiratory structures on plants and insects.


You have to remember what a lenticel is. Lenticels are openings in
plants that permit gas exchange. You probably do remember the leaf
structure called a stoma (Greek for mouth) which is bounded by guard
cells. The stoma is a leaf lenticel. This allows you to eliminate option
A. A trachea is a tube that conducts air and is used for both human and
insect structures, just as a siphon conducts water in aquatic animals, but
it is not the opening that was asked for. You can now eliminate options
C and E. While a mouth is an equivalent mammalian structure to a
lenticel, in insects the mouth is not used for gas exchange. This then
eliminates option D. The last choice is the one you would know if you
remembered the respiratory system of an insect.
3.

A. Questions 36 assay your understanding of the human diges-

tive system.
First you should identify the labeled structures. (I) is the parotid

AP BIOLOGY PRACTICE EXAM

51

gland; (II) is the liver; (III) is the stomach; (IV) is the gall bladder; (V) is
the pancreas; (VI) is the large intestine; (VII) is the small intestine; and
(VIII) is the appendix. Question 3 then asks you to identify the structures that produce digestive enzymes. The parotid produces amylase and
bicarbonate, the stomach produces pepsinogen, and the pancreas produces pancreatic juice, which contains a number of enzymes including
trypsinogen and lipase. The liver produces bile, which is stored in the
gall bladder, but neither of these two produces digestive enzymes.
4.

C. Glucagon in a hormone that helps regulate blood glucose levels.

It is released by the pancreas into the blood when glucose levels drop too
low. Glucagon stimulates the livers conversion of glycogen into glucose
and the production of insulin in the pancreas. As the glucose level rises
in the blood, the insulin stimulates the uptake of glucose by insulin-
dependent tissues.
5.

B. One of the functions of bile is to aid in the absorption of lipids.

Once a bolus of food moves from the stomach into the small intestine,
its acidity is neutralized and lipase begins to break triglycerides into
monoglycerides and fatty acids. These compounds are then emulsified
and packaged into spontaneously forming micelles. These micelles are
then absorbed into the endothelial cells of the small intestine, where
their contents are then routed to the lymph system for transport to the
circulatory system.
6.

E. Once food passes through the esophagus by peristalsis, it enters

the stomach through the cardiac sphincter. While there, it is bathed in


HCl and churned into a paste-like consistency. After about two hours
the food, now in the form of chyme, starts to move into the small intestine through the pyloric sphincter.
7.

A. The purpose of this question is to determine if you can use rea-

son to deduce the identification of a hypothetical biological entity.


All living organisms have a DNA-based genome, a membrane, ribosomes for translation, and RNA for transcription. Viruses may have a
DNA-based genome, may have a host-membrane derived envelope, may
be larger than some bacteria, may contain a stray ribosome from the

52

AP Biology

host cell, and may have a couple of enzymes. The key characteristic
differences between a virus and a living organism are that viruses contain either RNA or DNA, but not both, and while a virus may have an
enzyme or two, it lacks metabolic processes.
8. D. This question is designed to ascertain your understanding of the
functioning of mitochondria.
Chemiosmosis is the process whereby hydrogen ions are pumped
across a membrane. In the mitochondria, these proton pumps are powered by the energy extracted from electrons harvested from glycolysis
and the TCA cycle. Because of the gradient, the ions will rush back
through the membrane via a molecule of ATP synthase, thereby manufacturing ATP. If DNP is present, then the electron-transport system is
still functioning, and H+ ions are being pumped across the membrane,
but the leakage prevents the maintenance of the gradient required to
power ATP synthase. Thus, no energy is produced by the cell, and the
cell ceases to be able to maintain homeostasis.
9.

C. The purpose of this question is to determine the extent of your

understanding of pH and biological systems.


pH is a measure of the concentration of hydronium ions (H3O+) in
water. Distilled water has a neutral pH of 7.0 (where the H3O+ concentration matches the OH concentration).The scale is logarithmic, so
a tenfold drop in concentration raises the pH by one. Putting 1.0 ml
into 9.0 ml makes a total of 10.0 ml, so this represents a 1:10 dilution. Diluting an acidic solution makes it less acidic, so option A can be
removed from consideration. Making a pH 4.0 solution neutral would
require the addition of a basic solution, not just water, so E can also be
eliminated. A pH can always be measured, so option D can be removed.
A pH of 5.5 is halfway between 4.0 to 7.0 but really has no relevance.
Decreasing the concentration tenfold will raise the pH by 1.0.
10. E. This question is designed to determine your level of understanding of the flow of energy in photosynthesis.
Photons are packets of energy, not matter like electrons, so option
C can be ignored. Photosynthesis is conducted by autotrophs to

AP BIOLOGY PRACTICE EXAM

53

manufacture organic compounds for the storage of energy (electrons),


not to use the compounds, so option A can be eliminated. Since NAD
must actually be reduced by the capture of an electron, it lacks one to be
the originator, so option B is out as well. Chlorophyll only captures the
energy of a photon, which it then transfers to an electron. The electrons
come from a molecule of water in the process of photolysis, which also
releases hydrogen ions and oxygen gas when the electrons are harvested.
11. B. The purpose of this question is to determine the extent of your
understanding of membrane structure.
Chlorophyll in plants is contained within the thylakoid membranes of
the chloroplast, not the cell membrane, so option A is wrong. Both plant
and mammalian cell membranes contain ion channels and are attached
to the cytoskeletons, so options C and D are also wrong. Plants cells are
surrounded with a cell wall of cellulose, not a cell membrane of cellulose, so option E is wrong as well.
12. B. This question is based on one of the laboratory exercises and is
designed to see if you understand the differences between sexual and
asexual reproduction in fungi.
The ascomycetes produce their ascospores in the bag-like structures
called asci through the process of meiosis. This is why there is always an
even number of ascospores. Sporangiospores are asexually produced by
mitosis and thus are always present in larger, unpredictable numbers.
13. E. This question is designed to determine if you understand basic
mechanisms of evolution.
Analogous structures are those that have similar function but which
evolved separately within different groups of organisms. Homologous
structures are those that are evolutionarily linked by origin but may or
may not be similar in function. Options A, B, C, and D all posit analogous structures.
14. C. The purpose of this question is to determine your understanding of one of the mechanisms of the immune system.
The immune system has both innate and acquired mechanisms. The
purpose of immunizations against viral infections, bacterial infections,

54

AP Biology

and toxins is to activate one of the acquired mechanisms called memory.


When a person gets an infection, his or her body responds with the
production of protective antibodies. These antibodies are produced by
a subset of agranulocytes known as plasma cells, which are terminally
differentiated B-lymphocytes, or B-cells. Some of these B-cells, before
becoming plasma cells, will instead enter an arrested interphase and
stick around in the circulatory and lymph systems for decades, waiting
to become reactivated if required at a later time. Neurons have no direct
role in immunity, so option B can be discarded. While both platelets
and erythrocytes are found in the blood, they are associated with blood
clot formation and oxygen transport respectively, not immunity against
viruses. Neutrophils are very short-
lived cells only associated with
phagocytosis, so option D is also out.
15. D. Questions 1517 are designed to determine your understanding
of the major catabolic processes of cells.
First, you need to identify the processes described in the diagram.
Process (I) converts glucose to acetyl-CoA. The major portion of this
is glycolysis. Process (II) takes acetyl-CoA and harvests the high-energy
electrons contained in the cell, packing them onto NADH, and then
expels the waste gas carbon dioxide. This is the TCA cycle. Process (III),
if you have not guessed yet, is the electron-transport chain that takes
electrons and converts their energy into ATP. Neither options B nor E
are catabolic, and they can be discarded.
16. C. Identifying this as the electron-transport system is key to answering this question. Knowing that, what gas is required to act as the terminal electron acceptor? Neither chlorine, nitrogen, potassium, nor sodium.
17. D. To answer this question you need to remember the processes
involved. Both the electron-transport system (III) and the TCA cycle (II)
take place entirely within the mitochondrion. Glycolysis, the part that
goes from glucose to pyruvate, takes place within the cytoplasm, while
pyruvate conversion to acetyl-CoA takes place within the mitochondrion.
18. C. This question helps to ascertain your understanding of the phylogenetic classification of animals.

AP BIOLOGY PRACTICE EXAM

55

The simplest animals have no body cavities (acoelomates). These


include the poriferans, cnidarians, and platyhelminthes, thus eliminating options A and E. At the other extreme, the most advanced animals
have a coelom developed from a digestive tube (deuterostomes). These
include the echinoderms and the chordates, thus eliminating option B.
The nematodes are pseudocoelomates, thus eliminating option D.
19. A. This is another question to ascertain your understanding of phylogenetic relationships.
This is actually a true discovery. Of the eight regions that were recoverable, three most closely identified the dinosaur protein as bird-like.
Guess what that means: dinosaur probably tasted like chicken!
20. A. This question is designed to apply your understanding of biomolecular structure to derive relative sizes.
Carbon dioxide is a gas that consists of three atoms with a molecular
weight of 76 Da. A phospholipid consists of a phosphate group attached
to a glycerol head that is connected to two fatty acid chains of about 20
carbon atoms in length each for a total less than 30 carbon atoms; its
molecular weight is 700800 Da. DNA polymerase is an enzyme complex of between 50 and 150 kDa, or about 2S. The bacterial ribosome
consists of about 55 proteins plus rRNA strands that total 70S.
21. E. This question tests your ability to interpret data presented in
a graph.
A type I survivorship pattern is characteristic of large animals that
have few offspring in which they invest significant nurturing to ensure
longer lives. A type II survivorship pattern is characteristic of small animals that have moderate numbers of offspring in which they invest some
nurturing and that present a fairly even death rate throughout their lives.
A type III survivorship pattern is characteristic of very small animals
which produce huge numbers of offspring and invest no nurturing. This
produces extremely high death rates early on. The curve presented is
not one normally seen in a textbook, which usually presents the data as
mortality vs. maximum life span. This data simply shows a level death
rate, one that is fairly constant throughout the lifespan of the rodent.

56

AP Biology

22. E. Questions 22 and 23 are included to determine your understanding of a basic genetic-engineering procedure.
Restriction endonucleases are produced by bacteria as a defense
mechanism against bacteriophage attack. Each endonuclease cuts double-
stranded DNA at a specific sequence and usually cuts unevenly across
the two strands, producing sticky ends. These restriction enzymes are
identified by the first letter of the genus name and the first two letters
of the species name of the bacterium that produced the enzyme. These
three letters are followed by an additional unique identifier. Two such
common enzymes are Eco RI (for E. coli restriction enzyme I) and Hin
dIII (Haemophilus influenzae restriction enzyme dIII). Options A and C
can be eliminated because they are polymerases, as can B, which is also
called RNA-dependent DNA polymerase. Options D can be eliminated
because it is a ligase, not an endonuclease.
23. D. When DNA from any source is cut with the same restriction
enzyme, producing restriction fragments, then every sticky end is equally
complementary with every other sticky end. However, the cut in the
DNA backbone, produced by the endonuclease, is still present. This nick
in each strand must be rejoined, or ligated, by the enzyme responsible
for DNA repair caused by UV irradiation, DNA ligase.
24. A. This question is included to determine your understanding of
interspecies competition.
Sturgeons are primarily bottom feeders, while minnows prefer more
open water near shorelines. Since they rarely interrelate, their interaction is considered neutral; therefore option B can be discarded. The same
is true for option C. A rock is nonliving and simply serves as a substrate
for lichen attachment, and therefore no interspecies competition is
involved. Thus option D can be eliminated. While mistletoe is a parasite
on trees, this is not considered competition, as different resources are
involved (the tree draws from the soil, the mistletoe from the tree).
Woodpeckers are able to retrieve food from under the surface of wood,
while wrens are restricted to the surface only. The woodpecker is thus
able to exploit a food supply that the wren cannot.

AP BIOLOGY PRACTICE EXAM

57

25. C. This question is designed to allow you to indicate your level of


understanding of organization within a cell.
The nucleus is the location of the cells genome and the site of ribosomal subunit assembly, but not translation. Option B is thus eliminated. The Golgi complex, also called the Golgi apparatus, is the site of
protein modification and packaging, but not protein synthesis. Option D
is thus eliminated. No translation takes place on the cell membrane, so
option E is also eliminated. While the endoplasmic reticulum (ER) may
have newly synthesized proteins extruded into its lumen, no ribosomes
are present within the ER, although they sometimes are on its cytosolic
surface. Ribosomes are only present in the cytosol, which is the site of
all protein synthesis.
26. B. This question is here to determine your understanding of common inheritance patterns.
X-linked recessive means that the trait will be expressed in women
only if they are homozygous for the trait, but males will always express
the trait if they have it because they will be hemizygous. The mother is
heterozygous, because her father was afflicted and yet she is not. The
father is hemizygous for the trait, because he is afflicted. Constructing a
Punnett square shows that there is a 50 percent chance of a childs being
female, and these possibilities are not to be considered. Of the two possible male combinations one is afflicted, one is unafflicted. Therefore,
of possible male offspring, one-half will be afflicted and one-half will
be unafflicted.
27. D. The purpose of this question is to determine the extent of your
understanding of free energy in a cell.
Metabolism consists of all cellular processes used to process energy.
Catabolism consists of those reactions and pathways that break down large
molecules into smaller ones and release the energy therein contained.
Anabolism consists of those reactions and pathways that construct larger
molecules from smaller ones and store energy in the resulting additional
chemical bonds. Fatty acids are the major energy-containing substructures of triglycerides (lipids) that are broken down by b-oxidation to

58

AP Biology

produce acetyl-CoA, which, in turn, feeds into the TCA cycle, which
produces 11 ATP molecules from every acetyl-CoA fed in.
28. D. Questions 2830 are based on one of the lab exercises and
require you to interpret the diagram based on your understanding of leaf
structure and function.
There are two aspects to this question: which cells respire and which
cells photosynthesize. The ground tissue cells in the leaf do both. Dermal
tissue cells lack chloroplasts to allow light to penetrate into the leaf, so
they only respire. Vascular tissue cells do neither. The structure designated as (I) is cuticle and nonliving, so any answer including it can be
discarded; out go options A and B. The structures designated as (V) and
(VI) are dermal, so options C and E can be ignored.
29. B. Dermal tissues or structures of the leaf are those associated with
protection. This includes the cuticle (I), dermal cells (V), and stomata (VI).
30. E. Transpiration is the process during which water is allowed to
evaporate within the leaf and escape through the stomata. By acting like
a sink, the vanishing water draws more water (and nutrients) up from
the roots through the vascular system. Therefore both stomata (VI) and
xylem (included in IV) are involved in transpiration.
31. A. This question is designed to get you to deduce the mechanisms
involved in allergic reactions based on your understanding of the immune
system and cell-signaling pathways.
Mast cells contain membrane surface receptors that attach to a certain class of antibody called IgE. The presence of these allergen-specific
antibodies on the receptors sensitizes the mast cells to those allergens.
When the allergen appears, it attaches to the antibody, which changes
conformation and initiates a signal-transduction pathway. This results in
an influx of Ca2+ ions into the cell, which causes the release of histamine
into the tissue.
32. B. This question determines if you understand the mechanism of
translation and can apply that understanding.
In order for proteins to be produced, the coding must be brought to
the ribosome in the form of mRNA, even that required for viral proteins.

AP BIOLOGY PRACTICE EXAM

59

If the rabies virus genome is complementary to the mRNA, it must have


a mechanism for converting its complementary code into mRNA code.
Retroviruses do this by converting their RNA genome into DNA with
reverse transcriptase, then back into mRNA by normal transcription.
However, reverse transcriptase is not one of your choices. Options C and
E have nothing to do with transcription so can be removed as choices.
The enzymes in options A and D are directly involved in replication and
transcription directly, but do not solve the problem. This type of virus
comes packaged with its own RNA-dependent RNA polymerase inside
its capsid so it can immediately transcribe its complementary code into
mRNA as soon as it uncoats in the host cell.
33. C. Questions 3335 determine your abilities to interpret a graph
and apply your interpretation in order to explain the physiologic feedback mechanism.
This first question is rather easy. You know that diabetes is the condition of having elevated blood glucose levels. The normal values are
provided in the question, so you know diabetes would be a condition
when the value exceeds 150 mg/dl. The only curve that reflects a normal
resting value is III.
34. D. Diabetic shock occurs when excess insulin is in circulation. This
excess causes too much glucose to be moved from the blood into insulin-
sensitive cells. When the glucose level drops to the point where no cell
can get any glucose, then you are in insulin shock. While curves (II)
and (IV) both indicate excess insulin was administered, only the latter
remains critically low.
35. C. The curve designated (II) indicates an initial diabetic condition,
which means an insulin injection was administered at the time indicated
by the arrow. The glucose level plummeted, sending the person into
insulin shock. However, insulin shock can be treated by either removing
insulin from the blood (and its too late for that) or by increasing the
glucose level back up to the normal range. This is commonly done by
ingesting glucose in the form of a candy bar.
36. A. This is a question that simply requires shifting decimal places

60

AP Biology

and understanding metric measurements. It also demonstrates why blood


values are often expressed per deciliter, as opposed to the more commonly used per milliliter: because it requires fewer decimals. Converting
milligrams to grams shifts the decimal three positions to the left, and
converting deciliters to liters shifts the decimal one position to the right.
37. E. Questions 37 and 38 are designed to ascertain your understanding of the genetics and disease mechanism of a fairly common
perinatal problem.
Erythroblastosis fetalis is caused when an Rh mother becomes pregnant with a second or subsequent Rh+ child. During the birth of an
Rh+ child, an Rh mother gets exposed to the Rh factor antigens on the
infants red blood cells. Consequently, she starts to produce antibodies
against the Rh factor. If she later becomes pregnant with an Rh child,
the preformed antibodies will not damage the babys factorless red blood
cells. However, if she later becomes pregnant with an Rh+ child, her
antibodies will cross the placenta and attack the red blood cells of her
child. If severe enough, this can result in the death of the fetus.
38. E. Inheritance of the Rh blood group antigen is by simple dominance. A homozygous recessive mother lacking the antigen has to have
been exposed to the antigen during an earlier birth. The problem only
arises when a subsequent child inherits at least one copy of the gene that
produces the antigen.
39. C. This question is designed to see how well you understand the
relationships between two very common carbohydrates.
Glucose is central to the cellular metabolic pathways of most cells
and is manufactured within plants. However, it is primarily sucrose that
serves as the carbohydrate that transports energy in the sap of plants.
Combining a molecule of glucose with a molecule of fructose produces
a molecule of the disaccharide sucrose.
40. C. Questions 4042 are included to determine your grasp of the
mechanisms of competition with an ecosystem.
By looking at the graph you can see that only one population, designated I, was present at the beginning of the collection of data. The other

AP BIOLOGY PRACTICE EXAM

61

two appear to have been introduced into the ecosystem, either accidentally or intentionally. When comparing the curves designated I and II,
you can see an inverse relationship, i.e., as the numbers of II increase, the
numbers of I decrease, and vice versa. This suggests either that they are
direct competitors or that population II uses population I as either prey
or hosts. This latter is made less likely because when population I drops
to zero, population II continues to rise. Thus, it appears that population II
is a successful invasive species that completely outcompetes population I.
41. B. Population III arrives on the scene around week 45 and initially
prospers. At the same time, the invasive species numbers start to drop
off while population I starts to recover, making it appear that the drop
in population II was due to the introduction of population III. If that was
true, then it appears that population III was not capable of succeeding in
this new environment, and its numbers started to drop off, restoring the
growth of population II at the expense of population I. It appears likely
that population III was a natural predator of population II but could not
survive in this new environment.
42. C. If the understanding presented in the answer to question 41 is
correct, and it certainly appears to be the most reasonable of the choices,
then population I died out as a result of being outcompeted by population II. That the loss of population I was not due to either predation or
parasitism by population II is indicated by the latter still increasing in
numbers following the disappearance of population I. This most likely
means that population III failed independently of either I or II.
43. D. This question is included to determine your level of understanding of cell membrane function.
Essentially, the basic structure of eukaryotic and prokaryotic cell membranes is the same. They are the same thickness, eliminating option A. The
chemistry doesnt change from type to type, eliminating options B and C.
While option E is tempting, the role of the prokaryotic cell membrane is
more general and broader than that of the more specialized eukaryote.
44. A. This question allows you to demonstrate your ability to discern
selective pressures in evolutionary processes.

62

AP Biology

Venus flytraps live naturally only in areas with nitrogen-poor soils.


They grow slowly and within a rather limited range, so option E is out.
If the mechanisms that produced speciation were based on their speed,
as in option B, or the richness of their food, as in option C, or a wider
variation in nutrient forms, as in option D, then they would have a
much wider range and would be found in much larger numbers. It is
only because of their ability to survive in a rather hostile environment
that they exist at all.
45. A. The purpose of this question is to determine if you understand
and can identify symbiotic relationships.
Truffles, morels, and mushrooms are all sexual reproductive structures
of fungi, so options C, D, and E are eliminated immediately. Lichens are
fungal symbiotes, but with algae, not plants.
46. E. This question seeks to let you identify historical experiments
leading to the discovery that DNA contained genetic information.
Actually, the question is almost a giveaway. Option A identifies
phages, as in bacteriophages, as in bacteria, which dont have nuclei. So
option A can be discarded, as can B and C. Option D involves restriction
enzymes, but these all come from bacteria, as they serve to protect them
from phages. Even if you dont remember Acetabularia as a large nucleated cell, it is the only remaining choice.
47. C. This question allows you to demonstrate your understanding of
ABO blood group inheritance patterns.
Blood types A and B are determined by the presence of the A or
B allele, respectively. Blood type O is detected if neither A nor B is
present. A person with type A or B blood may be homozygous (AA or
BB) or heterozygous (AO or BO). Option A can be discarded because
type AB blood is not a certainty, with only a 25 percent probability.
Option B can be discarded because the explanation is wrong. Option E
can be discarded because the answer is irrelevant. While the reasoning
for option D is correct, the answer is wrong, so D is also thrown out. If
the parents are heterozygous, then any child has a 25 percent chance of
being homozygous for blood type O (OO).

AP BIOLOGY PRACTICE EXAM

63

48. B. The purpose of this question is to determine your level of understanding of basic evolutionary mechanisms.
Human DNA polymerase has an error rate of about one in 1107
bases. HIV replicates with an error rate of one in 2104 bases, or about
1000 times greater. This means that, on average, one of every two viruses
made will be mutations. While the vast majority will be noninfective or
nonproductive, the mutation rate ensures that the viral population will
constantly change. Option E is wrong because, while the explanation is
true, the numbers produced greatly speed the process. Options C and D
have the relationships reversed, i.e., the larger the genome, the greater,
not the less, the likelihood of error. Option A is wrong because, on average, 50 percent of all viruses produced will contain mutations.
49. D. Questions 4951 test your ability to identify the structures and
function of the urinary system.
The round globular structure (glomerulus) connected to the long
inverted loop (loop of Henle) should provide the key to identifying this
as a nephron, the functional unit of the kidney. The parallel tubes on
the left are renal arterioles and venules, and the tube on the right is the
collecting tubule.
50. A. If you know that vasopressin is another name for antidiuretic
hormone, then this helps, although even if you didnt know this you
could narrow down the choices. While the liver does secrete a few hormones, they are associated with growth and metabolism, not with the
kidney, so you can ignore option B. The hormones produced by the
hypothalamus and the adrenals are primarily developmental or growth
hormones, so options C and E can be ignored as well. To distinguish
between the anterior and posterior pituitary, just remember that the
posterior deals with the posterior (urine, uterine contractions).
51. A. Even if you didnt recognize the nephron from the image, you
can eliminate some choices here. You can see this is not the pituitary,
so you can eliminate option B. Because of the branching, you can eliminate the axons of neurons from being present, so option C can also
be removed from consideration. While the glomerulus resembles an

64

AP Biology

alveolus, the latter are always in clusters, so option D is out. The liver
has no such organized structure, so option E is similarly gone.
52. B. This question delves into your ability to identify evidence for
the endosymbiosis theory as a part of evolution.
The 70S bacterial ribosome consists of two subunits identified as
30S and 50S. The smaller subunit contains a strand of rRNA identified
as 16S. The rRNA is made by transcription in almost the exact same
way as mRNA, except the gene being transcribed is identified as 16S
rDNA. In humans, the 80S functional ribosomal subunit equivalents
found in the cytosol are 40S and 60S, with the rRNA in the smaller
subunit being 18S. However, the bacteria-sized mitochondria contain
their own self-replicating DNA and their own 16S rRNA as part of their
70S ribosome.
53. E. This question ascertains your understanding of and your ability
to identify the membrane transport mechanisms used by cells.
The question consists of two parts that need to be in agreement. If
you remember that this pump requires the expense of ATP, and thus the
pump is active transport, then you can eliminate options A, B, and D. If
you remember that the ions flow in opposite directions simultaneously,
and thus the pump is antiport, then you can eliminate options A, C, and
D. Combining the two leaves only one option remaining.
54. E. This question is here to determine if you can recognize a mechanism involved in evolution.
Option A can be ignored as it begs the issue: if a gene is fragmented
into repetitive parts, where did the repetitive parts come from? Option
B wont hold water, because there are hundreds of thousands of retrotransposons, while repetitive immunoglobulin sequences number in
the scores. Option C would not explain how the repetition came about
within introns. Option D is a poor choice because the concept is rather
Lamarckian, with phenotype driving genotype.
55. D. Questions 55 and 56 are designed to allow you to demonstrate
your understanding of energy transfer through an environment.
Light bathes the Earth constantly. Actually, a rather small percentage

AP BIOLOGY PRACTICE EXAM

65

is captured by photosynthesis, only about 1 percent. Therefore, if 1,000


kcal were available, only 10 kcal would end up in trophic level 1 biomass. While you might remember the exact percentage, the important
fact to remember is that the percentage captured is a very small amount
compared to the amount available.
56. B. While question 55 dealt with the amount of energy captured
from the available sunlight by consumers, this question deals with how
much of that which was captured is passed on to consumers at higher trophic levels. About 15 percent of producer biomass is converted to trophic
level 2; about 10 percent of that would be passed on to level 3. Thus, of
the original 10 kcal, 15 percent (or 1.5 kcal) would be transferred to level
2, and 10 percent of that (or 0.15 kcal) would be transferred to level 3,
for a total 16.5 percent transfer to higher trophic levels.
57. C. This question is designed to assay your level of understanding of
the role of hormones during pregnancy and your ability to reason from
two observations.
When a woman becomes pregnant, the production of LH and FSH
are suppressed, as they are associated with menstruation; thus option A
is removed from consideration. When a pregnancy begins, the process
is supported by the production of b-HCG, progesterone, and estrogen;
thus options B and D are no longer to be considered. Oxytocin is associated with perinatal and postnatal situations: the induction of labor
with uterine contractions and lactation. Thus option E is eliminated.
Rheumatoid arthritis is an autoimmune reaction caused by the infiltration of immune active cells into joints which causes damage. Thus, a
suppression of the immune system will reduce the attacks on the joints.
Genital warts are caused by infection with HPV, which the immune
system eventually suppresses, but if the immune system is suppressed,
the warts can reappear. When a woman becomes pregnant, her immune
response is slightly suppressed to prevent her from immunologically
rejecting the fetus.
58. A. This question determines if you can recognize the possible consequences of a genetic abnormality occurring during meiosis.

66

AP Biology

During meiosis, sister chromatids in synapsis, as indicated in the top


of the image, would undergo two sequential separation events. The correct process would be indicated if one chromosome of the original tetrad
remained in each nucleus. If this was mitosis, then only one division
would have been involved, so ignore option E. Crossing over occurs
before the reduction divisions shown, so ignore option B. Pleiotropy
deals with gene expression, not chromosomal separation, so option C
is also to be ignored. And it is a good thing that you worked on your
vocabulary so you immediately recognized that, although one of the
resulting cells may have lacked an X chromosome, this drawing has
nothing to do with hatred of women.
59. E. Nondisjunction is when the chromosomes separate unequally
during meiosis, a condition that would then be passed on to every cell
of a zygote that was produced by affected gametes. Knowing this allows
you to eliminate options A, B, and D. While you may not be able to
completely eliminate option C with what you know, the last choice
clearly indicates an extra X chromosome.
60. B. This question is designed to see if you are capable of applying
your understanding of the immune system to identify common elements.
During gestation, fingers and toes are initially all connected together
by a layer of skin. However, at some point the cells joining these structures receive a signal to undergo apoptosis, or genetically programmed
cell death. The loss of these cells then allows the independent functioning of these digits. All cells have the genes for the expression of
apoptosis. Cancer cells are allowed to proliferate only because of a loss
of apoptosis expression.
61. D. The purpose of this question is to allow you to demonstrate
your understanding of cell-surface receptors in viral transmission.
TMV is a rather common infector of many plant species, and people
who smoke or use tobacco products are often excluded from working
in plant nurseries for this reason. However, as with bacteriophages, just
because they are there doesnt mean you are doomed. TMV lacks the
ability to attach to and infect human cells.

AP BIOLOGY PRACTICE EXAM

67

62. C. This question is designed to get you to use your understanding


of plant anatomy to group phylogenetically.
What is pictured is a pine cone from a conifer, which is classified
within the gymnosperms, or vascular plants with naked seeds. Mosses
are nonvascular plants, so option D is out. Ferns and lycophytes are vascular but do not produce seeds, so you can also eliminate options A and
D from consideration. Monocots are a subset of angiosperms, or flowering plants, so they can also be eliminated. Of course, it would help if you
remembered that gingkos are also gymnosperms.
63. D. This is a question designed to get you to analyze probable function based on known structure.
Enzymes are able to act as biologic catalysts because of the near infinite variety of shapes which they are able to assume. Fatty acids are long
linear polymers of carbon and vary only occasionally as double bonds
appear, so out goes option B. Cellulose is a similar long polymer, but of
glucose, not just carbon, so option C is also out. DNA is very regular in
its structure as well, always in a long antiparallel double helix, so option
A is removed. ATP is too small, so you also get to eliminate option E.
Only RNA has the structural flexibility to become catalytic, in which
case it is known as ribozyme.
64. B. This question is included to get you to express your understanding of energy flow within a cell.
Electrons are transported to the electron system within the mitochondrion by molecules of NADH. NADH picks up most of its electrons
from the TCA cycle as it processes molecules of acetyl-CoA, and the
rest of them from the reactions that produce pyruvate. The electrons in
both acetyl-CoA and pyruvate come with them as they are processed
and released from a molecule of glucose. While cellulose is polyglucose,
it is not used as an energy source by plants, only as a structural polymer.
Therefore option D can be removed from consideration. ATP is an energy
carrier, not an electron source, so ignore option C as well. The electron
transport system discards its energy-drained electrons onto a molecule of
oxygen to produce water, so options A and E are not correct either.

68

AP Biology

65. E. This one deals with your understanding of energy flow in an


ecosystem not dependent on light.
Light cannot penetrate to these deep-sea vents so the populations
that arise are based on chemical energy sources. Although the water
temperature reaches several hundred degrees Celsius, it cannot boil
because of the high pressure. At that temperature, however, the water
easily dissolves the minerals from the crust, which immediately forms
large clouds of precipitate that produce the classic black smoker structure. Bacteria are able to harvest electrons from these precipitates and
grow to large numbers. They, in turn, are fed upon by protozoan and
larger planktonic organisms, which then support large animals such as
tube-worms and large crabs. Organisms cannot feed on heat so option
A is not true. Ecosystems always form from cross-dependent organisms
within a food web, so options B and D are not correct. Since no light can
penetrate to these depths there are no plants, so ignore option C.
66. A. This question centers on your ability to identify a basic evolutionary selection pressure.
Under completely equal conditions a phenotype that dies off three
times faster than another phenotype would quickly disappear from the
gene pool, so there must be some selective advantage to having bright
feathers. Options B, C, and D are probably true but do not explain the
survival of the bright plumage. Option E would actually probably have
the opposite effect of what is being observed. If females mated three
times more often with colorful males than with dull males, then the
bright phenotype would most likely be able to survive within the population in spite of the increased predation, simply because they would be
more reproductively successful.
67. E. Questions 6769 help you demonstrate your understanding of
the structure and function of a flower.
Meiosis is the process in which gametes are formed. In the case of
flowers, this means the formation of eggs and sperm. The eggs are produced in ovules and the sperm, as packaged in pollen, are produced in
the anther.

AP BIOLOGY PRACTICE EXAM

69

68. C. Flowers are pollinated by different means. Many are fertilized


by animals that are attracted to the flower by color or scent. Others,
which do not bother to invest their energy in producing large, bright,
or attractive flowers, instead invest in producing prodigious amounts of
pollen to be cast to the winds. To do this, they only need lots of pollen-
producing stamens as well as large stamens.
69. D. Simple fruit, like peaches, are formed from a single ovary.
Multiple fruit, such as pineapples, are formed from the fused ovaries
of multiple flowers. Accessory fruit, such as seen in the strawberry, are
formed from tissues not derived from the ovary. Aggregate fruit, such
as seen in blackberries and apples, are formed from multiple carpels in a
single flower.
70. A. This question is here to determine your ability to discriminate
the possible from the probable when it comes to climate change.
If global mean temperatures rise, then the oceans become warmer
and, as a result, are able to dissolve less oxygen, not more. This
eliminates option C from consideration. While a warming planet will
probably accelerate desertification, it will also open land previously
uninhabitable due to an excessively cold climate. This means the
amount of land available for cultivation will remain about the same,
so option D is out. Warm climes do not necessarily mean reduced animal diversity, as can be seen in the tropical rain forests, so option E
is removed as well. While a change in migration patterns is hard to
predict, it is more likely that flight lengths will increase, not decrease.
71. B. This question is here to allow you to use reasoning skills to identify a taxonomic group.
The building of the Aswan Dam reduced the river flow and allowed
a certain snail to migrate into the still waters. This snail serves as a host
for a trematode called Schistosoma. After infecting the snail, the fluke
changes form into a cercaria, which then infects humans wading in the
water by burrowing into the skin and migrating into the liver. There it
matures, mates, and lays eggs that are expelled from the body in feces or
urine, depending on the species. The key to this question is the term life

70

AP Biology

cycle. Bacteria do not have life cycles, only binary fission, so exclude
option D from your consideration. Similarly, arachnids do not metamorphose, so option A can be excluded. Since you had a lab on the physiology of the circulatory system using Daphnia as a model, and of course
you remember that water fleas are members of the order Cladocera,
then you can exclude option E as well. Option C, which includes squid
and octopi, can obviously be removed. Helminthes include tapeworms
(cestodes), roundworms (nematodes), and flukes (trematodes).
72. A. This question is designed to ascertain your level of understanding of cell communications.
Steroidal hormones are generally less specific than protein hormones,
and the immune system relies heavily on cell-surface receptors. Option
C can be discarded. Quorum sensing is bacterial, so option B is out.
Neurotransmitter release actually precludes cell-
to-
cell contact, so
option D is out. A change in ion flow through a membrane is a result of
cell signaling, not a form of it, so out goes option E as well.
73. E. Questions 7375 evaluate your ability to deduce the result of a
mutation based on your understanding of the regulation of gene expression and molecular biology techniques.
The lac operon is the archetype of the inducible bacterial gene. Under
normal conditions, lactose is not available to bacterial cells as a food
source because it is a rarely encountered carbohydrate. Consequently,
the operon that codes for the production of enzymes that metabolize
lactose is repressed. Under conditions of starvation and the appearance
of lactose, cells that possess this gene will be capable of using the newly
available lactose. The sugar enters the cell and binds to the repressor at
the operator region of the operon. The repressor then releases its hold on
the DNA, de-repressing (inducing) the operon. RNA polymerase then
binds to the promoter region, passes through the operator region, and
then transcribes the three structural genes that code for enzymes needed
to catabolize lactose. If the ampR gene was inserted into the sequence,
the polycistronic mRNA would be translated into the required enzymes
for lactose use, as well as the -lactamase enzyme coded by ampR.

AP BIOLOGY PRACTICE EXAM

71

However, the ability to resist the effects of ampicillin would only be seen
when the operon was induced by the presence of lactose. Thus options A
and C are incorrect and can be discarded. Option B makes reference to
tryptophan, but that is the archetypal repressible gene, not the inducible
one referred to here, so it can be discarded. Proofreading enzymes would
only work if there were a template available, and that would require a
diploid condition, so option D is out.
74. B. The evolution of a specific gene by random mutations, even
under tremendous selection pressure, would take an exceedingly lengthy
period of time, so option D is removed from consideration. The lac
operon and the tryptophan operon have nothing in common, so option
E is removed as well. If the genome of the bacterium was removed and
not replaced, how could the cell survive? Option C is also eliminated.
And, as tempting as option A might be, the bacterial genome is huge in
molecular terms and extremely difficult to manipulate as a whole, which
is why working with much smaller plasmids is so successful.
75. E. Three of these options are fairly easy to deal with. Option D is an
enzyme encoded within the lac operon, but is not a necessary part of the
exercise. Options A and C are both DNA polymerases, and the exercise
deals with transcription, not replication. Option B presents a transcription enzyme, but not the one needed in this exercise. The enzyme that is
necessary here is the one that allows researchers to cut-and-paste DNA.
76. A. This question is here to assess your understanding of the functions of various plant hormones.
IAA is an auxin that is responsible for promoting cell elongation.
Cytokinin stimulates cell division and retards senescence. Abscisic acid
induces dormancy. While you may want to avoid these because it is
gibberellin (option C) that you specifically are dealing with, the one you
really want to stay away from is 2,4-D, because, if you used it, it would
kill all your test plants. It is a herbicide.
77. C. The purpose of this question is to determine your level of understanding of the flow of oxygen through the body.
Both oxygen and carbon dioxide flow freely through cell membranes

72

AP Biology

in the direction of the concentration gradient. However, the direction of


the gradient throughout the body is reversed for the two gasses. While
the oxygen level is highest in the atmosphere, thus in the lung alveoli,
it is lowest in the tissues. This means the correct sequence for O2 is
(alveoli)pulmonary veinsleft atrium(left ventricle)(aorta)
(arteries)(arterioles)tissue

capillaries(venules)(veins)(vena

cava)(right atrium)right ventriclepulmonary arteries(back to


the alveoli). This question, however, is based on the reverse process of
ridding the body of CO2, which is highest in the tissues and lowest in the
pulmonary veins.
78. C. This question is included to determine your level of understanding of biological molecule structure.
There are four broad categories of biological molecules: carbo
hydrates, lipids, proteins, and nucleic acids. Almost everything within
cells is based on these or hybrid structures. Phenylalanine is an amino
acid, and proteins are polymers of amino acids. A person with phenylketonuria must minimize his or her intake of proteins and does not need to
exclude any of the other groups.
79. E. Questions 79 and 80 test your ability to recognize simple inheritance patterns from what appears to be complex data but is not really
that complex.
Each HLA antigen has numerous alleles, all codominant. Each sibling
received one of their mothers chromosomes and one of their fathers,
each of which has the HLA-A, HLA-B, HLA-C, and HLA-DR loci.
Because these genes are closely linked, they are commonly inherited
together. So the question is simply What is the probability of a siblings
having the same gene as another sibling? The answer is 25 percent. The
question asks, however, the probability of a fourth siblings matching
any one of the previous three, and for that you simply add up the probability of matching the first, or second, or third.
80. E. You will notice that when you compare the results for siblings
1 and 2, there are four results for the antigens present for HLA-B, -C,
and -DR, but only three for HLA-A. While this might be due to gene

AP BIOLOGY PRACTICE EXAM

73

deletion (option A) or lab error (option C), these are not likely and are
not necessarily explanations. Option B can be discarded, because this
would have caused the loss of all four antigens coded on the missing
chromosome, not just one. And option D can be discarded, because if it
affected one antigen, it would have affected all. The simplest answer is
that both parents have an allele for HLA-A1, and sibling 3 just happens
to be homozygous for that allele.
81. B. This question focuses on your understanding of the physiological response to an infection involving an immune-suppressing virus.
The incubation period for HIV is about two weeks. After this twoweek period, someone infected exhibits flu-like symptoms for a week to
10 days, after which the symptoms subside. The virus is replicating during this time and reaches a detectable level, as measured by viral antigen
level in the blood, at about the same time. Antibodies specific for HIV are
manufactured simultaneously but, because they are neutralizing viruses,
they do not reach detectable levels until about six weeks post-infection.
As the antibody levels rise, they are successful in clearing the blood of
any detectable viruses. The infection stays at this asymptomatic stage for
years. Eventually the virus starts to reduce the number of CD4+ T-helper
cells within the blood. As the antibody levels continue to decrease, the
virus starts to replicate at an uncontrollable rate, reducing the CD4+ cell
count to below 200 cells/mm3. When this threshold is crossed, the person is classified as having AIDS. While treatment with antiretrovirals can
keep the virus suppressed, which means infection with HIV is treatable,
the infection remains incurable, with a 100 percent fatality rate.
82. D. This question is included to determine your level of understanding of the structure and function of the chloroplast.
The energy of light is used in the process of photolysis as described
in the question. The electrons then flow through the electron-transport
mechanism embedded in the thylakoid membrane within the chloroplast. Within two complexes, photosystems I and II, energy from additional photons is harvested and transferred to the electrons, which are
eventually used to reduce NADP+ + H+ to NADPH. The H+ released

74

AP Biology

from photolysis then accumulates within the thylakoid space to drive


the synthesis of ATP by ATP synthase.
83. C. This question helps determine your level of understanding of a
very basic evolutionary concept.
The term fitness simply means the ability of an organism to survive and pass on its genes to its descendents. Lifespan is a rather poor
measure, because some very successful organisms that have been around
for millions of years have lifespans measured in hours, so option A is a
poor choice. Neither the number of chromosomes (bacteria lack them,
and yet they are very successful) nor the number of alleles has any value
in rating success, so out also go options B and D. Many animals once
covered the planet but are no longer with us, so option E is also a poor
choice. The only true way to measure fitness is by the ability to hang in
there during the millennia, which requires reproductive success.
84. A. This question attempts to clarify your understanding of the
molecular scales involved in measuring the cell.
Water surrounds everything associated with life, including enzymes,
so it is the smallest. Therefore you can discard options B, D, and E
because they do not list it first. Bacteria contain hundreds of different
enzymes, so an enzyme must be smaller than a bacterium, eliminating
option C. And yes, a bacterium is generally smaller than a chromosome.
85. D. This question measures your ability to apply what you know
about enzyme activity and your understanding of the digestive and circulatory systems to the interpretation of data presented in graphical form.
The curve designated I indicates the enzyme in question that has an
optimal enzyme activity at a pH of around 2. Now, where in the human
body do you find a pH that low? Correct, in the stomach. So enzyme I
is an enzyme secreted in the stomach. What about the curve designated
III with an optimal pH of about 8? Where do you find a pH that high?
Right again, in the duodenum, right after the release of bile from the gall
bladder. So enzyme III is a digestive enzyme secreted by the liver. While
curve II would fit just about any human enzyme having an optimal activity near physiological pH (7.4), that is especially true in the blood. If

AP BIOLOGY PRACTICE EXAM

75

you remember that pepsin is secreted as pepsinogen in the stomach, you


can eliminate options B, C, and E. If you remember that chymotrypsin
(the chyme is a clue) is associated with protein digestion in the small
intestine, then you can eliminate options A, B, and E. There is only
one option not eliminated with this reasoning. You do not even need to
remember that fibrin is associated with blood-clot formation to get this
one right.
86. E. This question centers on your ability to discern the implications
of the ring based on your understanding of fungal sexual reproduction.
Toadstools and mushrooms are both the basidiospore-bearing structures formed when the subsurface hyphae of two different mating types
intersect with each other. At each location where such contact is made
the thallus arises out of the soil. A ring indicates that a colony of one
mating type started growing in the middle of another mating type and
the ring shows the intersection of the mating types.
87. A. Questions 8789 focus on your understanding of inheritance
and gene expression.
The easiest way to approach this question is by discarding the obviously wrong answers first. By remembering that hemophilia is a lethal
disorder, you understand that if a person bleeds to death it can hardly
increase longevity, so options B and D can be discarded. By remembering
that a person is afflicted if they inherit two recessive alleles (if female) or
one (if male) you can discard option C, because it is wrong. Disruptive
selection is when the selective pressures remove the mean and encourage the extremes; unfortunately, the extreme here is death, so option E
is out as well. While the usual examples of the founder effect center on
geographic separation, here the separation is social.
88. C. While all of the options are plausible, and some actually produce
known morbidity, in this case the disorder is caused by a lack of factor
VIII in the blood. It is the only cause of hemophilia that is sex linked.
89. C. The purpose of this question is to determine your level of understanding of a form of asexual reproduction known as parthenogenesis.
First of all, some reptiles in an all-female populations can lay eggs that

76

AP Biology

hatch normally, so option A is incorrect. While sperm can be stored for


later use in many animal species, the phenomenon does not occur in
reptiles, so option E is out. While some animals such as mollusks and
fish can change gender, it does not happen in reptiles. Parthenogenesis
involves the ability of some cells to interrupt the meiotic process early,
leaving some diploid cells to continue as a form of zygote. This does,
however, produce a population that continues to be only female.
90. B. This question is designed to determine your understanding of
the Calvin-Benson cycle.
In the 1940s this very experiment was conducted by Melvin Calvin
and Andrew Benson using the newly developed techniques of radiolabeling. While you might be tempted to think the technique would
not work because the boiling water would destroy the radiolabeled
materials, this is not the case, as only proteins are so labile. And, since
you remember the Calvin-Benson cycle, you remember that RuBP
fixes atmospheric CO2 to form two molecules of PGA. This is the
technique they used to figure that out because PGA was where the 14C
first showed up.
91. E. This question is included to determine if you are able to deduct
the correct answer based on your understanding of simple metabolism.
The amount of oxygen released by photosynthesis is greater than that
consumed by organic organisms. If that werent the case, there would be
no residual oxygen in the atmosphere for us, so you can exclude options
A and C. Even if Hydrilla had air bladders, they would contain air, not
pure oxygen, so option C can be excluded as well. While option B gets
closer to the actual mechanism, it is not fish that consume the oxygen, it
is the bacteria that come later.
92. D. Questions 9294 ascertain your understanding of the female
reproductive cycle and its control by hormones, and your ability to interpret data in a graphical form.
The key here is to use two benchmarks on the graph: menstruation
starting on day 0 and the FSH peak on day 14. Remembering that FSH
and LH are tied together also helps a lot. Given that, you can see that

AP BIOLOGY PRACTICE EXAM

77

curve I represents LH, which eliminates options B, C, and E. While you


could select randomly from the last two options and possibly improve
your score, remembering that the level of progesterone rises during
the latter parts of the secretory phase of the uterine cycle gives you
the answer.
93. B. Deducing the hormones from the previous question helps to
answer this one, but remembering the major glands of the endocrine
system and the hormones they secrete helps a lot as well. In addition,
remembering that the hypothalamus controls the pituitary by neural
mechanisms and secretes nothing on its own allows you to eliminate
options that include it: A, D, and E. Since the pituitary provides most
of the coordinating hormones, and the spike around day 14 for both LH
and FSH synchronizes both the uterine and ovarian cycles, then you can
conclude that option C is incorrect.
94. A. The key here is to connect the mechanism of follicle rupture
with the more common term ovulation. The surge of FSH (follicle
stimulating hormone) triggers ovulation. The surge of LH (luteinizing hormone) supports the formation of the hormone-secreting corpus
luteum. The date of ovulation usually falls between the thirteenth and
sixteenth day from the onset of menses.
95. A. This question is designed to determine your level of understanding of basic membrane structure.
The fluid mosaic model is based on the concept of proteins floating in a hydrophobic interface between the two aqueous environments
found inside and outside the cell. It has nothing to do with motility,
eliminating options D and E, nor with the movement of substances
through the membrane, eliminating option C. While the cytoskeleton
does provide for cell structure, it has no fluid qualities. When antibodies bind to membrane proteins scattered over the surface, the cell will
move and collect those bound receptors together into one region of the
cell surface. This process can be observed in a UV microscope, as the
fluorescent material all congregate into one spot, forming a cap on one
portion of the cell.

78

AP Biology

96. D. This question focuses on your understanding of the organization


and function of the brain and the central nervous system.
Speech is a fine motor skill, so option B can be ignored. A sulcus is
a division between regions of the brain, not a functional unit of it, so
option E can be removed from consideration. The occipital lobe, at the
back of the brain, is associated with vision, so option C is out. Most of
the parietal lobe is associated with sensory interpretation, so option A
is out. The frontal lobe is best associated with memory, which includes
how to speak.
97. B. This question concentrates on your ability to extrapolate
between what you know and what you probably dont know.
So many words, so little time. You may know that these all deal with
the means of bearing young in animals. But even if you didnt, an understanding of classical root words and suffixes would be of great benefit
here. The root vivi-(as in vivacious or vivisection) means alive. The
root ov-(as in ovary or ovum) means egg. And the modifier par- means
to hold or to bear. Knowing this you can determine the answer.
Viviparity means bearing young alive. Oviparity means bearing young
in eggs. Ovoviviparity means having young in eggs that hatch internally,
producing a live birth. Placental viviparity is what humans have: live
birth from a placenta. While you may get hung up on the last term, just
remember the question: which is the oldest? In evolutionary terms, animal life developed from multicellular organisms producing fertilization
of eggs outside the body; therefore eggs came first, and it doesnt matter
what matrotrophy means.
98. C. This question is designed to reveal the depth of your knowledge
of basic organic chemical reactions.
Carbohydrate monomers are also known as simple sugars. Sugars connect together to form a glycoside bond. These bonds are formed when
OH groups on adjacent molecules expel a molecule of water and join
together through the remaining atom of oxygen. When H2O is removed
from a molecule, it is called condensation; therefore this mechanism
is called condensation synthesis. Knowing this allows you to discard

AP BIOLOGY PRACTICE EXAM

79

options A, B, and D. Of the remaining two options, anabolic is an adjective meaning building up, the opposite of what you are looking for.
99. C. This question ascertains your recollection of the repertoire of
plant responses.
The root word photo-refers to light, so out goes option B. The root
grav-you can readily link to gravity, so out goes option E. The root
seismo-, as in seismograph, can be linked to motion, so you can also eliminate option A. The Greek root somn-means sleep and ambu- means
moving, so this term means sleep walking, which is not a plant option.
The Greek root thigmo-means touch, and trop-means growth, so the
word means growth in response to what it is touching.
100. B. This question helps to determine your level of understanding of
the organisms used in biologic research.
While we use the binomial system of identifying organisms to the genus
and species level, we also frequently use shortened forms which everyone
involved is just supposed to know. Sometimes just using the first letter
of a genus name is common, so E. coli is generally more recognizable
than the full genus name. Since this is a bacterium, it cannot be used in
learning about cellular differentiation, so option D can be ignored. You
should recognize at least the genus name Daphnia, which was used in
your lab on physiology, not on gene expression, so you can skip option E.
While Drosophila might be tempting, and has been used in learning gene
expression, its strength has been in understanding inheritance patterns,
so option A is out of the running. Pisum you probably remember from
when you studied Mendel because this was the variety of pea he used
in his research, so eliminate option C as well. Although the last one is a
mouthful, you may well recognize C. elegans, the nematode that lives for
less than three weeks and has exactly 959 cells as an adult.

Section II: Free-Response Questions


Please note that the answers provided here are only typical of what
might be acceptable. Many free-response questions have several possible
answers. Just remember that is important for you to answer each portion

80

AP Biology

of the question reasonably. Note also that the answers need not be lengthy,
just complete. It is helpful to use the correct terminology in answering
these questions, as this is commonly rewarded in the scoring rubric. You
do not need to provide introductory or concluding paragraphs.
1. A. One example of positive control of gene expression is the inducible operon in bacteria, of which the lac operon is the best example.
Under normal conditions, lactose is not available to the cell and the
operon is repressed. However, under periods of starvation when lactose
becomes available, the lactose binds to the repressor and causes it to
release its hold on the operator region of the DNA. This allows the RNA
polymerase to bind at the promoter region, pass through the operator
region, and transcribe the genes needed for the use of lactose as an energy
source. The resulting mRNA is translated into the needed proteins.
One example of negative control of gene expression is the repressible operon in bacteria, of which the trp operon is the best example.
Under normal conditions of cell growth the amino acid tryptophan is
required for the manufacture of proteins. Consequently, the gene is normally expressed and the enzymes needed for tryptophan synthesis are
transcribed and the mRNA is translated. However, under conditions of
no growth, additional tryptophan is not needed. Under these conditions,
excess tryptophan binds to an inactive repressor, causing it to become
active and bind at the trp operons operator region. This represses the
operon and shuts down tryptophan synthesis.
B. One positive control mechanism of enzyme activity is the
requirement for a cofactor or coenzyme. Cofactors and coenzymes
bind at an enzymes allosteric site, causing a conformational change in
the enzyme structure. This change converts the enzyme from an inactive to an active form, allowing the enzyme to catalyze the reaction in
which it participates.
One negative control mechanism of enzyme activity is the presence of either competitive or noncompetitive inhibitors. A competitive
inhibitor interferes with the ability of reactants to enter the active site

AP BIOLOGY PRACTICE EXAM

81

of the enzyme. The more competitive inhibitor there is in solution, the


slower the effective rate of enzyme activity. A noncompetitive inhibitor
binds at an allosteric site and changes the conformation of the enzyme,
changing it from an active to an inactive form. The more noncompetitive inhibitor there is in a solution, the slower the effective rate of the
enzyme until saturation is reached, at which point additional inhibitor
produces no increasing effect.
2. A. Blood pressure in mammals is controlled by the autonomic nervous system which consists of two parts, the sympathetic and the parasympathetic nervous systems. Most organs function only within a narrow
range of blood pressure, so both excessively high and excessively low pressures must be avoided. The autonomic nervous system controls the tension
of the smooth muscles surrounding the arteries and veins. Increasing the
vascular pressure is simply a matter of relaxing or increasing the muscle
tension. Relaxation of the muscles increases the lumen diameter, increasing the volume available for the blood, and drops the pressure. Increasing
the muscle tension decreases the lumen diameter, decreasing the volume
available for the blood, and raising the blood pressure.
B. Blood pressure can also be controlled by altering the fluid balance between the tissues and blood by controlling the Na+ level in the
blood, which is controlled significantly by the readsorption into the
blood from the urine within the kidneys. The more Na+ is in the blood,
the more water will be drawn into the blood to maintain proper Na+
levels, which increases blood volume and, in turn, raises blood pressure.
When blood pressure drops, the liver secretes angiotensin into the blood,
which eventually becomes a vasoconstrictor, raising blood pressure, and
which also stimulates the secretion of aldosterone. Aldosterone from the
adrenal glands promotes the readsorbtion of Na+ into the blood, also
raising blood pressure.
C. Chronic high blood pressure damages the glomerulus in the kidneys and causes kidney failure. A weakened area of a blood vessel may
burst if subjected to excessive blood pressure.

82

AP Biology

3. A. One mechanism of prezygotic isolation is habitat isolation. This


occurs when two populations become separated geographically or by
habitat choice. An example of habitat isolation is species restrictions to
certain levels of a rain forest which prevent interactions between similar
organisms. Another mechanism of prezygotic isolation is behavioral isolation. This occurs when mating takes place only after specific recognition of the opposite sex of the same species. A good example of this is
the flashing light patterns of fireflies, unique for each species.
B. One mechanism of postzygotic isolation is hybrid sterility. This
occurs when two species can mate but the offspring are infertile. An
example of this is the mating of a horse with a donkey, which produces
an infertile mule. Another mechanism is gamete isolation. This occurs
when the sperm of one species cannot physically fertilize the egg of
another. An example of this is when the sperm of one species cannot
survive in the reproductive structures of another.
4. A. Vascular plants have to transport water and nutrients from the
soil to the leaves sucrose from the leaves back to the rest of the plant.
The vascular tissues that accomplish this are xylem and phloem, respectively. Xylem is composed of vessel elements and tracheids. Phloem is
composed of sieve-tube members and companion cells.
Water and nutrients are transported to the leaves by transpiration.
Water within the soil enters into the root, flowing between the cells
until it reaches the Casparian strip, at which point it flows within cells
until it reaches the xylem. This produces some pressure that pushes the
water up from the roots. Additionally, evaporation within the leaves,
controlled by the guard cells of the stomata, removes water from the top
of the water column, which, because of the cohesive characteristic of
water, pulls the water up as well.
Once photosynthesis has taken place and sucrose synthesized, it is
transported from the leaves to the rest of the plant by a mechanism
identified as the pressure-flow model. Sugar is loaded into the phloem
by active transport. The increase in sugar concentration then pulls water

AP BIOLOGY PRACTICE EXAM

83

into the phloem by osmosis. This increases the pressure within the tube,
forcing the sugar (or sap) to flow downward. At the other (sink) end of
the plant, tissues are pulling sugar out of the phloem, and water follows
out as well. This decreases the pressure, sucking the sap downward.
B. Both xylem and phloem are structurally supported by adjacent
parenchymal cells of the ground tissue system. This is required because a
break in either system would eliminate vital fluid flow through that system. Additionally, these vessels and supporting structures are organized
into strengthened vascular bundles interspersed in the pith. A second
protective structure includes the dermal tissue system that strengthens
the plant from the outside. The woody bark protects the vascular cambium from damage, and the cork cambium protects the vital tissues by
filling in fissures caused by the increase in girth during secondary growth.

You might also like